2
$\begingroup$

Let $\eta$ be a continuous bounded function on $(0, \infty)^{2}$ so that $\eta(0,0)=1$. Let $A$ be a bounded operator on $\ell^{2}(\mathbb{Z}_{\geq 0})=\ell^{2}$ (by bounded operator I will always mean such an object) and let $A_{j,k}$ be its "matrix entries" with respect to the standard basis of $\ell^{2}$.

Suppose now that for any integer $N\geq 1$ and any bounded operator $A$, the infinite matrices $A^{(N)}$ with entries $$A^{(N)}_{j,k}=\eta(\frac{j}{N}, \frac{k}{N})A_{j,k}$$ form a uniformly bounded sequence, meaning that there exists $C>0$, independent of $N$, so that $$\|A^{(N)}\|_{op}\leq C \|A\|_{op},$$ where $\|\cdot\|_{op}$ denotes the usual operator norm.

It is very easy to show that under these assumptions, one can have uniform convergence of $A^{(N)}\to A$ as $N\to \infty$ (i.e. $\|A^{(N)}-A\|_{op}\to 0$) whenever $A$ is a finite matrix (as we have entrywise convergence). By density of the finite matrices and an $\varepsilon/3$-argument it is not so difficult to show that the same happens whenever we restrict to the ideal of compact operators.

What can we say about the convergence of $A^{(N)}\to A$ for an infinite matrix? Of course, the best one can hope for, dropping the compactness of $A$, is convergence in the Strong Operator Topology, but I cannot find an argument that proves my statement. Any help would be appreciated!!!

$\endgroup$
2
  • $\begingroup$ I am confused. What is your $\ell^2 $, sequences or matrices? $\endgroup$ Mar 27, 2018 at 17:56
  • $\begingroup$ @FedorPetrov the matrix $A$ acts on $\ell^{2}$. $\endgroup$
    – Raphael
    Mar 28, 2018 at 13:55

1 Answer 1

4
$\begingroup$

By the uniform bound on $\|A^{(N)}\|$ and linearity, SOT convergence follows from the $\ell^2$-norm convergence, for every $i$, of the $i$-th column $A^{(N)} e_i$ to the $i$-th column $A e_i$. This convergence is straightforward (say by the dominated convergence theorem).

$\endgroup$
2
  • $\begingroup$ Thank you for your answer! However, would you mind being more specific? I have tried this approach and I get stuck when trying to prove that $$\|A^{(N)}e_{j}-Ae_{j}\|_{2}\to 0$$ as $N \to \infty.$ It is clear to me that all the entries of the columns of $A^{(N)}$ converge to those of $A$, however convergence in $\ell^{2}$-norm doen't seem immediate. Could you give more details? I am very sorry for asking this dumb question. $\endgroup$
    – Raphael
    Mar 28, 2018 at 14:00
  • $\begingroup$ @Raphael As I wrote, this follows from the dominated convergence theorem. You want to prove that $\sum_i |A^{(N)}_{i,j}-A_{i,j}|^2$ tends to zero. You know that each term in the sum goes to zero. But also that the $i$-th term is less than $(1+\|\eta\|_\infty)^2 |A_{i,j}|^2$, which is summable as a function of $i$. $\endgroup$ Mar 28, 2018 at 16:57

Your Answer

By clicking “Post Your Answer”, you agree to our terms of service and acknowledge you have read our privacy policy.

Not the answer you're looking for? Browse other questions tagged or ask your own question.